If Roamin' is the second program, then each of the following could be true EXCEPT:

Jaimee-Salgado on October 3, 2018

Can you please explain?

Can someone explain this question and the correct answer?

Replies
Create a free account to read and take part in forum discussions.

Already have an account? log in

Mehran on October 16, 2018

@Jaimee-Salgado if R is the second slot, it would be at either 1:30 or 2, depending on whether G is first.

But can R actually appear at 1:30? Only if S T or W appeared at 1.

We know that T can't be at 1pm because of rule 2.

We know that S can't be at 1pm because R is shown before S according to rule 3.

We also know that if W is shown before T, which would be the case if W was at 1pm, T must be immediately after W, which would mean T would be at 1:30pm which can't be true.

This means that R can't appear at 1:30pm, which means the following scenario must result:

1 G G
2 R

Now let's take a look at (D), which would put W at 2:30pm as follows:

1 G G
2 R W
3 T S

Notice that W at 2:30pm would invoke rule 4, which means T must be shown immediately after W. That would place T in 3pm, which VIOLATES rule 2 that requires T to start on the half hour rather than the hour.

So (D) cannot be true so (D) would be the correct answer.

Hope that helps! Please let us know if you have any other questions.

Fatou on January 14, 2020

Hey,
I understand all of the answers except A. How can Sundown be the third program, when R_S and T has to start on the half hour, and If W_T then, it would force the WT block. So the only variable left is G to take over the 1 hour block. What am I missing?

Thank you

lklop on November 29, 2020

For (D) how come it can't be GG, R, S, W, T? wouldn't that work and allow T to be fifth? It also follows all the rules as far as I can tell.

Emil-Kunkin on January 18 at 06:01PM

There is no rule that says r must start two before s. The scenario iklop described would not be D, so I'm not sure what his comment was but future readers can ignore